Endo/Repo Final

Pataasin ang iyong marka sa homework at exams ngayon gamit ang Quizwiz!

A 35-year-old G0 woman comes to the office because of six months of spotting between her periods and a desire for a pregnancy. She reports using 30 pads/cycle the last two months and has blood clots and cramping pain. Prior menses were light and required 15 pads/cycle. She has been trying to conceive for six months. Her work-up included a transvaginal ultrasound which revealed a 2 cm endometrial polyp. What is the next best step in the management of this patient? A. Hysteroscopic polypectomy B. Observation C. Combination birth control pills D. Endometrial ablation E. In-vitro fertilization

A. Management of an endometrial polyp includes the following: observation, medical management with progestin, curettage, surgical removal (polypectomy) via hysteroscopy, and hysterectomy. Observation is not recommended if the polyp is > 1.5 cm. In women with infertility polypectomy is the treatment of choice. While her inability to get pregnant may be more complicated than just her polyp, removal of the polyp should occur prior to infertility treatments.

A 27-year-old G1P0 woman complains of mood swings and fatigue in the week prior to her menstrual period. These symptoms have worsened over the past six months. Some months the symptoms are so severe she misses several days of work. Her medical history is otherwise unremarkable and a physical examination is normal. Which of the following is the most appropriate next step in this patient's management? A. Symptom diary for two months B. Dietary changes C. Anxiolytic agent D. Psychiatric consultation E. Pelvic ultrasound

A. Obtaining further history with a menstrual calendar determines the cyclic nature of the PMS or PMDD symptoms and helps guide appropriate therapy. While dietary changes may help, it is first important to establish the diagnosis. An anxiolytic agent or psychiatric consultation is not indicated.

A 16-year-old female goes to the doctor to discuss why she has not had a menstrual cycle. She is healthy and plays weekend volleyball. She studies hard and gets good grades in school. She has a good relationship with her parents. On examination she is 5 feet 1 inch tall and weighs 80 pounds. Breast and pubic hair growth are at a Tanner stage II. External genital examination is normal. What is the most likely reason this patient has not had any menses? A. Inadequate body weight B. Poor nutrition C. Inadequate sleep D. Excessive exercise E. Familial reasons

A. This patient does not weigh enough since a body weight of 85 to 106 pounds is needed before menses begins. There are two other critical elements for secondary sexual characteristics: sleep and optic exposure to sunlight. These factors especially can delay the onset of menarche. Psychosocial causes of delayed puberty include eating disorders, excessive exercise (playing volleyball on the weekend is not considered excessive exercise), and stress or depression.

A 14-year-old G0 female reports menarche six months ago, with increasingly heavy menstrual flow causing her to miss several days of school. Three months ago, her pediatrician started her on oral contraceptives to control her menstrual periods, but she continues to bleed heavily. Her previous medical history is unremarkable. The patient has a normal body habitus for her age. Appropriate breast and pubic hair development is present. Her hemoglobin is 9.1 mg/dl, hematocrit 27.8%, a urine pregnancy test is negative. Which of the following etiologies for menorrhagia is most likely the cause of her symptoms? A. Uterine leiomyoma B. Thyroid disorder C. Coagulation disorder D. Endometrial hyperplasia E. Chronic endometritis

C. Disorders of clotting may present with menstrual symptoms in young women, with Von Willeberand disease being most common. Leiomyomas typically present in women in their 30's and 40's. Endometrial hyperplasia can occur in younger anovulatory patients, but the short duration of this patient's symptoms makes this less likely. She does not have any signs of infection or thyroid disease.

A 19-year-old G0 woman presents with severe menstrual pain which causes her to miss school. She takes 600 mg of ibuprofen every four to six hours to control the pain, but this does not relieve the discomfort. She is sexually active, with one partner (she has had two lifetime partners) and uses condoms for contraception. Examination is normal. What is the most appropriate next step in the management of this patient? A. Copper IUD B. GnRH agonist C. Oral contraceptives D. Continuous medroxyprogesterone E. Laparoscopy

C. Dysmenorrhea or painful menstrual cramps is often incapacitating. Oral contraceptives will not only relieve primary dysmenorrhea, but also provide more reliable contraception. Copper IUD's have the potential to cause heavier and more painful periods. Continuous oral Medroxyprogesterone may be effective, but will not provide contraception. Depo-Provera would be a better choice. GnRH agonists are too expensive and have too high a side effect profile to be used for this purpose.

A 15-year-old female mentions to her doctor that she has never had a menstrual cycle. She is healthy, active in school activities and eats a normal diet. She denies ever being sexually active. On physical examination, she has Tanner stage II breast and pubic hair growth, and average weight and height. Vaginal opening is present and appears normal. What is the most appropriate next step in her management? A. Pelvic ultrasound B. Oral contraceptive pills C. Reassurance D. MRI of sella turcica E. Cortisol challenge test

C. Normal age for menarche is between nine and 17. Since this patient has secondary sexual characteristics and normal anatomy, she should be offered reassurance that she is normal and her menses will probably start soon.

A 31-year-old G3P0 woman presents with amenorrhea for six months. She is otherwise in good health and is not taking any medications. She had a miscarriage seven months ago, which was complicated by an infection and required antibiotics and a dilation and curettage procedure. Her examination is normal. Her laboratory results show a Beta-hCG <5 mIU/mL, and normal TSH and prolactin levels. What is the most likely underlying cause of this patient's amenorrhea? A. Chronic endometritis B. Recurrent miscarriages C. Hypothalamic-pituitary amenorrhea D. Asherman's syndrome E. Sheehan's syndrome

D. Asherman's syndrome can be caused by curettage or endometritis. The intrauterine synechiae or adhesions result from trauma to the basal layer of the endometrium, which causes amenorrhea. Chronic endometritis may be associated with abnormal uterine bleeding and not amenorrhea. Hypothalamic amenorrhea is unlikely because of the temporal relationship of her amenorrhea to the procedure. Sheehan's syndrome is typically due to severe postpartum hemorrhage leading to pituitary apoplexy.

A 58-year-old G3P3 woman has been postmenopausal for five years and is concerned about osteoporosis. She has declined hormone therapy in the past. Her mother has a history of a hip fracture at age 82. A physical exam is unremarkable. In addition to weight bearing exercise and vitamin D supplementation, what optimal daily calcium intake should she take? A. None B. 400 mg C. 800 mg D. 1200 mg E. 1600 mg

D. Calcium absorption decreases with age because of a decrease in biologically active vitamin D. A positive calcium balance is necessary to prevent osteoporosis. Calcium supplementation reduces bone loss and decreases fractures in individuals with low dietary intakes. In order to remain in zero calcium balance, postmenopausal women require a total of 1200 mg of elemental calcium per day.

A 23-year-old G0 woman presents to the office because she has not had any menses for four months. She has a long history of irregular menstrual cycles since menarche at age 14. She is otherwise in good health and is not taking any medications. She is thin and has chronic anxiety (BMI 16). Her Beta-hCG is < 5 mIU/mL, and her prolactin and TSH levels are normal. What would be the next best diagnostic test to order? A. Estrogen level B. Progesterone level C. Gonadotropin releasing hormone level (GnRH) D. Follicle stimulating hormone (FSH) E. Dehydroepiandrosterone sulfate (DHEAS)

D. The causes of hypothalamic-pituitary amenorrhea are functional (weight loss, obesity, excessive exercise), drugs (marijuana and tranquilizers), neoplasia (pituitary adenomas), psychogenic (chronic anxiety and anorexia nervosa), and certain other chronic medical conditions. In this case, the next step to make a diagnosis is to obtain FSH which would be expected to be in the low range. You already know that her prolactin level is normal, which is consistent with the diagnosis. Prolactin would be elevated with a prolactin-secreting pituitary adenoma.

An 18-year-old G0 woman presents with a one-year history of hirsutism and acne. She had menarche at age 14 and her menses have been irregular every 26-60 days. Her sister has a similar pattern of hair growth. The patient is 5 feet 4 inches tall and weighs 180 pounds. On exam, a few terminal hairs were identified on her chin and upper lip. TSH, prolactin, total testosterone, and DHEAS levels are normal. Which of the following is the most appropriate next test to evaluate this patient's condition? A. Estradiol B. Serum cortisol C. Urinary cortisol D. Random blood glucose E. 17-hydroxyprogesterone

E. Checking 17-hydroxyprogesterone would rule out late onset 21-hydroxylase deficiency. Normal TSH, Prolactin, total testosterone and DHEAS levels rule out pituitary or adrenal tumors. The patient could have polycystic ovarian syndrome; however, normal serum testosterone levels make it less likely. Blood glucose would not help determine the etiology of hisrsutism.

1cm

How many cm must a thyroid nodule be in order to take FNA

steroids, then hypotension and hyperK

Tx for secondary adrenal insufficiency?

cushing adrenal adenoma

acth is elevated in ___ but decreased with ______

RAIU

after nodule on thyroid is found and TSH is low, next step

U/s (then FNA)

after nodule on thyroid is found and TSH is low. If RAIU shows cold, next step

neonatal thyrotoxicosis

goiter, tachypnea, tachy, cardimegaly, diarrhea, poor weight gain within 1-2 days of birth, even if mom had thyroidectomy (still has maternal IgG)

addison

hypoadrenalism is also called

1cm

if nodule on thyroid and u/s shows it, if it is > ____ cm, do FNA, if it is smaller, repeat the U/s

iodide

medicine to give in thyroid storm

MRI brain

next step if ACTH is increased wth cushing

tsh

next step if nodule on thyroid

u/s

nodule on thyroid + normal or high TSH, next step

conn

primary hyperaldosteronism, adrenal adenoma

sestamibi

scan to find struma ovarii

conn

sx of htn, hyperna, hypoK, metabolic alkalosis, high aldosterone, low renin

papillary

thyroid cancer that is most common, associated with radiation

papillary

thyroid cancer with psammoma bodies

anti-tpo, ama

2 ab present in hashimoto

amiodarone, lithium

2 meds that cause low thyroid

lithium, methoxyflurane, demeclocycline

3 meds that cna cause nephrogenic DI

A 27-year-old G0 woman comes to the clinic because she has been unable to conceive for the last year. She is in good health and has not used any hormonal contraception in the past. She had normal cycles in the past every 28 days until about six months ago. At that time, she began to have irregular menses every two to three months, with some spotting in between. She is not taking any medications. She has no history of abnormal Pap smears or sexually transmitted infections. Her physical examination is normal. Laboratory tests show: Results Normal Values Results Normal Values TSH 10 mIU/ml 0.5-4.0 mIU/ml Free T4 0.2 ng/dl 0.8-1.8 ng/dl Prolactin 40 ng/ml <20 ng/ml FSH 6 mIU/ml 5-25 mIU/ml LH 4 mIU/ml 5-25 mIU/ml What is the most appropriate step in the management of this patient? A. Begin levothyroxine B. Begin bromocriptine C. Order a clomiphene citrate ovulation challenge test D. Obtain a brain MRI E. Order a thyroid ultrasound

A

A 26-year-old G0 woman presents with hirsutism and irregular menses. Her mother, who is diabetic, had similar complaints prior to menopause. On physical exam, this patient is noted to have terminal hair on her chin and a gray-brown velvety discoloration on the back of her neck. This lesion is acanthosis nigricans. Which of the following is the most appropriate first test to order for this patient? A. Fasting insulin B. TSH C. 17-hydroxyprogesterone level D. Cortisol level E. Pelvic ultrasound

A. Acanthosis nigricans is associated with elevated androgen levels and hyperinsulinemia. Since this woman has a family history of diabetes and also has acanthosis nigricans, the most appropriate test of those listed would be the fasting insulin. The other tests would also be reasonable, but hyperinsulinemia is most likely in this patient.

A 37-year-old G0 woman complains that she experiences mood swings, irritability, bloating and headaches monthly for two to three days prior to her menstrual cycle. Her medical history is unremarkable and physical examination is normal. The physician advises her to keep a calendar of her symptoms. He also recommends a balanced diet, avoidance of caffeine and alcohol, and daily regular exercise. The patient has never exercised regularly and wonders how this will help her mood swings and bloating. Which of the following would provide the best explanation for the benefits of exercise on her PMS symptoms? A. Endorphins B. Cortisol C. Progesterone D. Estrogen E. Androgen

A. Exercise increases circulating endorphins in the brain which are "feel good" hormones and act similar to serotonin. Therefore, in addition to being a benefit to cardiovascular health, regular exercise can significantly decrease symptoms of PMS.

A 51-year-old G1P1 woman presents for annual examination. She notes vaginal dryness, some hot flashes, and fatigue. She reports that her last menstrual period was 14 months ago. She and her husband use lubrication for intercourse, and she denies any significant pain. Her past medical history is significant for hypertension, which she controls with diet and regular exercise. She is concerned that she should begin hormone replacement, because her mother started HRT around the same age. Which of the following is the most appropriate next step in her management? A. Expectant management B. FSH level C. LH level D. Estrogen level E. Initiate combination HRT

A. Expectant management is reasonable in this patient, as she notes minimal menopausal symptoms. Her vaginal dryness is not interfering with her ability to enjoy intercourse, and she has only occasional hot flashes. An FSH level is not indicated as by definition she is menopausal given amenorrhea for greater than 12 months. Estrogen and LH levels are not indicated in the diagnosis of menopause. While HRT is appropriate for patients with significant menopausal symptoms, and should be used at the lowest effective dose for the shortest amount of time, this patient at this time is not experiencing significant symptoms and therefore HRT should not be initiated at this time.

A 35-year-old G2P2 woman comes to the office due to heavy menstrual periods. The heavy periods started three years ago and have gradually worsened in amount of flow and duration. The periods are now interfering with her daily activities. The patient had two spontaneous vaginal deliveries. She smokes one pack of cigarettes per day. On pelvic examination, the cervix appears normal and the uterus is normal in size, without adnexal masses or tenderness. A urine pregnancy test is negative. TSH and prolactin levels are normal. Hemoglobin is 12.5 mg/dl. On pelvic ultrasound, a 2 cm submucosal leiomyoma is noted. An endometrial biopsy is consistent with a secretory endometrium; no neoplasia is found. Which of the following would be the best therapeutic option for this patient if she desires to have another child? A. Hysteroscopy with myoma resection B. Laparoscopic myomectomy C. Endometrial ablation D. Oral contraceptives E. Dilation and curettage

A. Hysteroscopic myomectomy preserves the uterus, while removing the pathology causing the patient's symptoms. A laparoscopic approach is not indicated as the myoma is submucosal and not accessible using a laparoscopic approach. Endometrial ablation destroys the endometrium and can create Asherman's syndrome, thus it is reserved for patients who have completed childbearing. Dilation and curettage is unlikely to remove the myoma and is a blind procedure (carried out without direct visualization). Oral contraceptives would typically help with heavy menses, but are contraindicated in this patient, who is over 35 and smokes.

A 27-year-old G0 woman presents to the clinic because of concerns that she has not been able to get pregnant for the last three months. She married a year ago and was using condoms for contraception, which she stopped three months ago when she decided to start a family. She is in good health and her only medication is a prenatal vitamin. Her periods are regular every 28 days with normal flow; her last period was two weeks ago. She has no history of sexually transmitted infections and no abnormal Pap smears. Her husband is also healthy with no medical problems. She is 5 feet 4 inches tall and weighs 130 pounds. Her examination, including a pelvic exam, is completely normal. What is the most appropriate next step in the management of this patient? A. Reassurance and observation B. Hysterosalpingogram C. Transvaginal pelvic ultrasound D. Semen analysis E. Mid-cycle blood LH and FSH levels

A. Reassurance and observation is most appropriate as the patient has only been trying to conceive for three months. After one month, 20% of couples will conceive; after three months, 50%; after six months, 75%; and after 12 months, 90% will conceive. Primary infertility is defined as the inability to conceive for one year without contraception. The patient is young and healthy with no obvious reasons for infertility, so at this point reassurance and observation is the proper management. There is no reason to order any studies now, especially since she has normal cycles.

A 17-year-old female is brought to the physician because she has never had a menstrual cycle. She has normal breast and pubic hair development. Physical examination reveals a small vaginal opening with a blind pouch. Pelvic ultrasound reveals normal ovaries, but absence of uterus and cervix. Which of the following is the most appropriate next study in this patient? A. Renal ultrasound B. Brain MRI C. FSH and LH determination D. Cortisol level E. Testosterone level

A. Renal anomalies occur in 25-35% of females with Mullerian agenesis. The uterus and cervix are absent, but the ovaries function normally and, therefore, secondary sexual characteristics are present. You would expect the karyotype in this patient to be 46,XX and testosterone levels in the female range.

A 17-year-old G0 female presents with hirsutism, irregular menses and obesity. Her mother is moderately obese with mild hirsutism. Recently, the patient's hirsutism has worsened and she has been depressed. She has also gained 20 pounds in the past two months and has noticed stretch marks on her abdomen. At the time of your examination, you note that she has terminal hair growth on her chin and hair growth on the back of her hands. Her cheeks appear flushed. Her stretch marks are purplish in color. The rest of her exam is normal. Which of the following is the most appropriate first test to order for this patient? A. Overnight dexamethasone suppression test B. 17-hydroxyprogesterone C. Fasting insulin D. TSH E. Pelvic ultrasound

A. Since Cushing's syndrome is suspected, either a dexamethasone suppression test or a 24-hour urinary measurement for cortisol can be performed. Elevated cortisol would be indicative of Cushing's syndrome. The other tests listed would be reasonable, but only after Cushing's syndrome had been excluded.

A 21-year-old G0 woman comes to the office because of acne, irregular menses and hirsutism. She initially was evaluated six months ago, at which time she was diagnosed with idiopathic hirsutism. She was started on oral contraceptive pills to improve her symptoms. Menstrual periods now occur every month, but her hirsutism has not significantly improved. In addition to the oral contraceptives, which of the following would be an appropriate treatment for hirsutism? A. Spironolactone B. Lupron C. Danazol D. Depo-Provera E. Steroids

A. Spironolactone, an aldosterone antagonist diuretic, can also be used in addition to the oral contraceptives for hirsutism. Danazol is primarily used for the treatment of endometriosis and may actually worsen hirsutism and acne. Lupron and Depo-Provera are also reasonable as second-line treatments of hirsutism, had the patient not already been on oral contraceptives. Steroids will not help.

A 54-year-old G2P2 woman presents to your office for a health maintenance examination. Her last menstrual period was eight months ago. She complains of severe vasomotor symptoms, vaginal dryness, and dyspareunia, and desires treatment for her symptoms. She has otherwise been in good health and has no significant past medical or surgical history. Her family history is significant for a mother who has severe osteoporosis at the age of 75, a grandmother who died of breast cancer at the age of 79, and a father who died at age 77 from a myocardial infarction. She denies smoking, alcohol or drug use. On physical exam her BP is 130/78, pulse is 84, and BMI is 26. The remainder of her exam is within normal limits except for severe vaginal atrophy noted on the pelvic examination. The best recommendation for this patient would include which of the following? A. Lowest effective dose of combination hormone replacement therapy for the shortest duration possible B. Long term hormone replacement therapy to treat her vasomotor symptoms and prevention of osteoporosis C. Testosterone cream D. Progesterone cream E. Biosphophonates

A. The American College of Obstetricians and Gynecologists (ACOG) recommendations on hormone replacement therapy considers hormone replacement therapy (HRT) the most effective treatment for severe menopausal symptoms that include hot flashes, night sweats and vaginal dryness. The physician should counsel the woman about the risks and benefits before initiating treatment. ACOG recommends "the smallest effective dose for the shortest possible time and annual reviews of the decision to take hormones." HRT should not be used to prevent cardiovascular disease due to the slight increase in risk of breast cancer, myocardial infarction, cerebrovascular accident, and thromoboembolic events. A woman with an intact uterus should not use estrogen-only therapy because of the increased risk of endometrial cancer. In addition to the same risks as FDA approved treatments, bioidentical hormones such as testosterone and progesterone cream may have additional associated risks. While her family history is significant for one second degree relative with breast cancer, this is not an absolute contraindication for short-term HRT.

A 58-year-old G3P3 woman who has been menopausal since age 50 comes to you for a health maintenance examination. She is in good health, eats a balanced diet, exercises regularly, and has an unremarkable physical exam. Her bone mineral density as determined by central dual energy X-ray absorptiometry is -1.7. She wants to discuss treatment for her osteopenia. What is the next step in the management of this patient? A. Evaluate her risk factors for fracture B. Determine her frequency of exercise C. Assess her exogenous dietary intake of estrogen D. Assess her exogenous dietary intake of progesterone E. Repeat DEXA scan in one year

A. The World Health Organization (WHO) defines osteopenia (low bone mass) as -1 to -2.5. The American College of Obstetricians and Gynecologists (ACOG) Committee Opinion recommends that physicians interpret T scores between −1.0 and −2.5 in combination with the patient's risk factors for fracture. The authors state: "Clinicians must be careful because the diagnosis of osteopenia often is interpreted as indicating a pathologic skeletal condition or significant bone loss, neither of which is necessarily true. Until better models of absolute fracture risk exist, postmenopausal women in their 50s with T scores in the osteopenia range and without risk factors may well benefit from counseling on calcium and vitamin D intake and risk factor reduction to delay initiation of pharmacologic intervention." Some of the risk factors for fracture include prior fracture, family history of osteoporosis, race, dementia, history of falls, poor nutrition, smoking, low body mass index, estrogen deficiency, alcoholism, and insufficient physical activity.

A 24-year-old G1P1 woman presents with a complaint of decreasing breast size and hirsutism noted over the last three months. She also notes her skin feels oily and her husband has mentioned her voice seems to be getting deeper. She has no medical or surgical problems and takes no medications. Physical examination reveals oily skin, upper lip and chin terminal hair, and normal appearing breasts. Pelvic examination reveals her clitoris to be 2 cm in length and 1 cm wide. Which of the following is the most likely cause of her symptom constellation? A. Steroid cell tumor B. Hypothyroidism C. Polycystic ovary syndrome D. Congenital adrenal hyperplasia (early onset) E. Anabolic steroids

A. The likely cause of this patient's sudden onset of symptom is an increase in androgens due to a tumor. Hirsutism is often the result of a benign condition, however, may be a sign of significant disease if sudden in onset and coupled with virilization. Virilization in the female may be manifested by frontal hair thinning, oily skin or acne, deepening of the voice, clitoral enlargement, menstrual irregularities, and increased muscle strength. Possible causes of virilization include PCOS, hypothyroidism, androgen producing tumors (ovarian, adrenal, or pituitary), and anabolic steroid use. A rare cause may be late onset congenital adrenal hyperplasia.

A 34-year-old G2P2 woman presents with concerns of hormonal changes. She is worried about facial hair growth, worsening acne, and deepening of her voice. She also realized that she has missed her period for two months, and has been sexually active and had tubal ligation. On examination, she is moderately obese and noted to have severe acne, upper lip and chin terminal hair. Her abdomen is obese with moderate hair growth. Pelvic examination is most notable for an enlarged clitoris, and pelvic exam reveals an enlarged right-sided adnexal mass. Which of the following is the most likely diagnosis in this patient? A. Sertoli-Leydig cell tumors B. Granulosa cell tumor C. Benign cystic teratoma D. Thecoma E. Cystadenoma

A. The most likely diagnosis in this patient is a testosterone-secreting ovarian tumor. Sertoli-Leydig cell tumors are commonly diagnosed in women between the ages of 20-40, and are most often unilateral. Rapid onset of hirsutism and virilizing signs are hallmarks of this disease, and include many of the findings in this patient including acne, hirsutism, amenorrhea, clitoral hypertrophy, and deepening of the voice. Abnormal laboratory findings include suppression of FSH and LH, marked elevation of testosterone, and presence of an ovarian mass. The constellation of findings is most consistent with a testosterone-secreting tumor, and a pelvic ultrasound will confirm the presence of an ovarian mass. The other tumors do not cause virilization. Granulosa cell tumors and thecomas are estrogen-secreting tumors.

A 53-year-old G2P2 woman comes to your office complaining of six months of worsening hot flashes, vaginal dryness, night sweats, and sleep disturbances. Her last normal menstrual period was six months ago and she has been experiencing intermittent small amounts of vaginal bleeding. Her medical history is significant for hypertension, which is well-controlled by a calcium-channel blocker, type 2 diabetes, for which she takes metformin, and hyperthyroidism, for which she takes propylthiouracil. The patient is 5 feet 7 inches tall and weighs 140 pounds. Blood pressure is 120/70. Physical examination is unremarkable. Which of the following medical conditions in this patient is a contraindication to treatment of menopausal symptoms with hormone therapy? A. Vaginal bleeding B. Hypertension C. Diabetes D. Osteoporosis E. Hyperthyroidism

A. The principal symptom of endometrial cancer is abnormal vaginal bleeding. Although the patient's worsening symptoms make treatment an important consideration, the specific organic cause(s) of abnormal bleeding must be ruled out prior to initiating therapy. A tissue diagnosis consistent with normal endometrium or a pelvic ultrasound with an endometrial stripe of <4 mm ought to be documented. In addition, risks and benefits of hormone replacement therapy must be discussed with this patient at length prior to beginning treatment.

A 15-year-old G0 female presents with severe menstrual pain for the past 12 months. The pain is severe enough for her to miss school. The pain is not relieved with ibuprofen 600 mg every four hours. She is not sexually active and the workup reveals no pathology. Which mechanism of action best explains why oral contraceptives would be the most appropriate treatment for this patient? A. Inducing endometrial atrophy B. Decreasing inflammation C. Increasing prolactin levels D. Decreasing inhibin levels E. Thickening cervical mucous

A. The progestin in oral contraceptives causes endometrial atrophy. Since prostaglandins are produced in the endometrium, there would be less produced. Dysmenorrhea should be improved.

A 42-year-old G4P4 woman presents with a history of progressively worsening severe menstrual pain. Menses are regular, but she complains of very heavy flow requiring both a menstrual pad and tampon and often bleeds through both. She takes oxycodone that her husband used for back pain to relieve her dysmenorrhea. She had a tubal ligation four years ago. Pelvic examination shows an enlarged, soft, boggy uterus. No masses are palpated. Pregnancy test is negative, hemoglobin 9.8 and hematocrit 28.3%. What is the most likely diagnosis? A. Adenomyosis B. Endometrial carcinoma C. Endometriosis D. Primary dysmenorrhea E. Endometrial hyperplasia

A. This is a typical presentation of adenomyosis (presence of endometrial glands and supporting tissues in the muscle of the uterus). The gland tissue grows during the menstrual cycle and, at menses, tries to slough, but cannot escape the uterine muscle and flow out of the cervix as part of normal menses. This trapping of the blood and tissue causes uterine pain in the form of monthly menstrual cramps. Endometrial hyperplasia and carcinoma are less likely in a woman with regular menses and no inter-menstrual spotting. Endometriosis would most likely have presented earlier in life and would not explain the enlarged uterus.

A 17-year-old G0 female is brought in by her mother because she has not yet had any menses. She is otherwise in good health, but recently has been experiencing cyclical lower abdominal cramping. She has never had sexual intercourse. She is 5 feet 6 inches tall and weighs 120 pounds. On examination, her breasts are Tanner Stage IV. She has some suprapubic tenderness on abdominal exam. Her pelvic exam reveals normal external genitalia, but there was difficulty inserting a speculum due to patient's discomfort. Beta-hCG < 5 mIU/mL. What is the most likely diagnosis in this patient? A. Genital tract outflow obstruction B. Müllerian agenesis C. Hypothalamic-pituitary dysfunction D. Psychogenic amenorrhea E. Constitutional delay in menarche

A. This patient's primary amenorrhea, with normal secondary sexual characteristics, development and cyclical abdominal pain, points to an anatomical cause of amenorrhea, which is preventing menstrual bleeding. An imperforate hymen commonly causes this and the treatment is surgical. In Mϋllerian agenesis, or Mayer-Rokitansky-Kϋster-Hauser syndrome, there is congential absence of the vagina and usually an absence of the uterus and fallopian tubes. Ovarian function is normal and all the secondary sexual characteristics of puberty occur at the appropriate time.

A 22-year-old G0 woman college student returns for follow-up of mood swings and difficulty concentrating on her schoolwork the week before her menses for the past 12 months. Her past medical history is unremarkable and physical examination is normal. Which of the following would be an appropriate treatment option for this patient? A. Oral contraceptive pills B. Reassurance and observation C. Methylphenidate (Ritalin) D. Gabapentin E. Ginkgo

A. This woman has premenstrual syndrome (PMS) with symptoms that warrant treatment. Patients with PMS and premenstrual dysphoric disorder (PMDD) experience adverse physical, psychological and behavioral symptoms during the luteal phase of the menstrual cycle. PMS is characterized by mild to moderate symptoms, while PMDD is associated with severe symptoms that seriously impair usual daily functioning and personal relationships. Mild symptoms of PMS often improve by suppressing the hypothalamic-pituitary-ovarian axis with oral contraceptive pills. Ritalin and Ginkgo are not effective treatments for PMS. Gabapentin is used for neuropathic pain and will not help alleviate her symptoms.

A 4-year-old female is being evaluated for premature hair growth in the pubic area. She has no breast development and has not had any menstrual bleeding. Laboratory evaluation revealed high DHEA and DHEAS levels and low levels of LH and FSH. Which of the following is the most likely cause of this patient's premature adrenarche? A. Idiopathic isosexual precocious puberty B. Congenital adrenal hyperplasia C. Hypothalamic dysfunction D. Pituitary adenoma E. Polycystic ovarian syndrome

B. Congenital adrenal hyperplasia of the 21-hydroxylase type results in the adrenal being unable to produce adequate cortisol as a result of a partial block in the conversion of 17-hydroxyprogesterone to desoxycorticosterone, with the accumulation of adrenal androgens. This leads to precocious adrenarche. Treatment includes steroid replacement. Idiopathic isosexual precocious puberty is GnRH dependent and leads to an appropriate (although early) order of pubertal events. Some girls with premature adrenarche develop polycystic ovarian syndrome in adolescence, but not at this age.

A 49-year-old G2P2 woman status post hysterectomy at age 45 for fibroids presents to your office complaining of severe vasomotor symptoms for three months. Hot flashes are affecting her quality of life and she would like to discuss options for treatment. What treatment option for hot flashes associated with menopause do you recommend as the most effective? A. Lifestyle modifications such as dressing in layers B. Estrogen C. Selective estrogen receptor modulator (SERMs) D. Selective serotonin reuptake inhibitors (SSRIs) E. Treatment with phytoestrogen (soy)

B. Except for estrogen receptor modulator therapy, all of the above treatment options will improve hot flash symptoms. Treatment with estrogen is most effective, and the current recommendation is for the lowest dose for the shortest duration of time. Hot flashes will resolve completely in 90% of patients receiving this therapy. Raloxifene, a selective estrogen receptor modulator, may actually cause hot flashes to worsen in a patient who has not stopped having these symptoms completely. SSRI antidepressants, some anti-seizure medications and alternative treatments, such as soy products and herbs, have not been shown to be as effective as estrogen.

A 16-year-old female comes to the doctor to discuss contraception. She recently became sexually active and states she has never had a menstrual cycle. She regularly attends school and participates in the band. On physical examination, she is 5 feet 3 inches tall and weighs 130 pounds. She has no secondary sexual characteristics with normal appearing external genitalia. The physician suspects Kallmann syndrome. Which of the following diagnostic tests will help confirm the diagnosis? A. MRI of the pituitary B. Olfactory challenge C. Measurement of testosterone levels D. Pelvic ultrasound E. Cortisol levels

B. Kallmann syndrome is characterized by olfactory tract hypoplasia and the arcuate nucleus does not secrete GnRH. Therefore, these females have no sense of smell and do not develop secondary sexual characteristics. The diagnosis is often one of exclusion found during the workup of delayed puberty. The presence of anosmia with delayed puberty should suggest Kallmann syndrome. Treatment is pulsatile GnRH therapy. Testosterone levels would be needed if the patient had symptoms of male hormone (androgen) production such as excess hair growth, male pattern baldness, or clitoris enlargement. Cortisol levels are obtained if you suspect adrenal or pituitary gland problems.

A 19-year-old G0 woman presents with severe menstrual pain that causes her to miss school. She takes 600 mg of ibuprofen every four to six hours to control the pain, but this does not relieve the discomfort. You started oral contraceptives, but her symptoms persisted. She also tried Depo-Provera for three months without much improvement. She still has menstrual pain and continues to miss some classes. What is the most appropriate next step in the management? A. Transdermal narcotic for pain relief B. Diagnostic laparoscopy C. Presacral neurectomy D. Prescribe a selective serotonin reuptake inhibitor E. Prescribe GnRH agonist with estrogen add-back

B. Laparoscopy is recommended to confirm the diagnosis of endometriosis and exclude other causes of secondary dysmenorrhea. SSRIs are not used to treat dysmenorrhea, rather they are a good treatment for PMS. Some authors suggest that a course of GnRH agonists are appropriate, with laparoscopy reserved for those women who have pain during or after completion of a three-month course, but using estrogen add-back would remove the diagnostic sensitivity of the GnRH agonist.

A 28-year-old G0 woman presents for an annual examination. She is in good health and not taking any medications. She had a history of normal cycles until six months ago, when she stopped having menses after starting an intense exercise regimen. She is 5 feet 6 inches tall and weighs 120 pounds. Her examination is completely normal. Her urine pregnancy test is negative. What is the underlying pathophysiology of the disease process in this patient? A. Psychogenic amenorrhea B. Premature ovarian failure C. Hypothalamic amenorrhea D. Androgen excess E. Hyperthyroidism

C. Amenorrhea associated with exercise falls under the category of hypothalamic amenorrhea, which causes chronic anovulation. Although it may be related to energy requirements, alterations in the hypothalamic-pituitary-ovarian axis have been described in athletic women. The patient's history and physical exam make ovarian failure, androgen excess and hyperthyroidism less likely, although a TSH level would still be recommended.

A 32-year-old G2P2 woman is concerned about symptoms associated with her menstrual cycle. During the second half of her cycle, she feels anxious, sad and has difficulty sleeping. She has done research on the Internet and believes she suffers from premenstrual dysphoric disorder (PMDD). Which of the following symptoms of the patient is most consistent with this diagnosis? A. Cyclic constellation of symptoms during the follicular phase B. Cyclic occurrence of a minimum of described symptoms and interference in social functioning C. Chronic, mild depressive symptoms that have been present for many years D. Depressed mood or the loss of interest or pleasure in activities E. Anxiety/nervousness interfering in social functioning

B. PMDD is a psychiatric diagnosis, describing a severe form of premenstrual syndrome in which the diagnostic criteria include five out of 11 clearly defined symptoms, functional impairment and prospective charting of symptoms present during the last week of the luteal phase that begin to resolve with the beginning of the follicular phase. All three areas of symptoms need to be represented for the diagnosis of PMDD.

A 32-year-old G0 woman presents with irregular menses occurring every six to eight weeks for the past eight months. The bleeding alternates between light and heavy. Her irregular menses were treated successfully with medroxyprogesterone acetate (MPA), 10 mg every day, taken for 10 days each month. By which mechanism does the MPA control her periods? A.Stimulates rapid endometrial growth and regeneration of glandular stumps B. Converts endometrium from proliferative to secretory C. Promotes release of Prostaglandin F2α D. Regenerates functional layer of the endometrium E. Decreases luteal phase inhibin production

B. Patients with anovulatory bleeding have predominantly proliferative endometrium from unopposed stimulation by estrogen. Progestins inhibit further endometrial growth, converting the proliferative to secretory endometrium. Withdrawal of the progestin then mimics the effect of the involution of the corpus luteum, creating a normal sloughing of the endometrium. Stimulation of rapid endometrial growth, conversion of proliferative to secretory endometrium, and regeneration of the functional layer describe effects of estrogen on the endometrium. Inhibin is increased in the luteal phase.

An 18-year-old G0 woman comes in for a health maintenance examination with her mother. The mother had severe PMS symptoms in her twenties and thirties and would like to know if her daughter would inherit this as well. Which of the following has the strongest association with premenstrual syndrome? A. Obesity B. Positive family history C. History of early menarche D. Insulin dependent diabetes mellitus E. Vitamin K deficiency

B. Risk factors for PMS include a family history of premenstrual syndrome (PMS) and Vitamin B6, calcium, or magnesium deficiency. PMS becomes increasingly common as women age through their thirties, and symptoms sometimes get worse over time. Previous anxiety, depression or other mental health problems are significant risk factors for developing premenstrual dysphoric disorder (PMDD). There is no known association between premenstrual syndrome and obesity or insulin dependent diabetes mellitus.

A 20-year-old G0 woman presents with severe menstrual pain. She takes 600 mg of ibuprofen every four to six hours to control the pain, but this does not relieve the discomfort. She is sexually active with one partner and has four lifetime partners. She uses condoms for contraception. Past medical history is unremarkable, except for breast cancer in her father's sister. Examination is normal. Which of the following is the most appropriate test for this patient? A. HPV DNA typing B. Chlamydia testing C. Pap smear D. Hysterosalpingogram E. Diagnostic laparoscopy

B. The US Preventive Services Task Force recommends chlamydia and gonorrhea screening for all sexually active patients, age 25 and younger. Since pelvic inflammatory disease is a cause of secondary dysmenorrhea, it needs to be evaluated as a potential cause of her symptoms. Although HPV screening is common, it can be used as an adjunct to cytology in primary screening in women 30 years or older, and is not indicated in a 20 year old. Pap smears are not indicated in women under 21 years of age regardless of sexual history. A hysterosalpingogram is used for infertility work-up and will not necessarily help determine the cause of her pain. A diagnostic laparoscopy would be premature at this point.

A 9-year-old female goes to the doctor's office for a regular check-up. She is healthy, active in school sports and gets good grades. On examination she is 4 feet 8 inches tall and weighs 80 pounds. She is concerned about when she might expect to have her first menstrual period since her friends have been talking about it. On physical examinations, she has Tanner stage 1 breasts and no pubic hair. You explain to her that she can expect to experience which of the following first sexual developments? A. Adrenarche B. Thelarche C. Growth spurt D. Menarche E. Pubarche

B. The normal and predictable sequence of sexual maturation proceeds with breast budding, then adrenarche (hair growth), a growth spurt and then menarche. In a minority of cases, pubarche (pubic hair growth) can occur before thelarche (breast/areolar development). Breast development begins around the age of 10 and average age of menarche is 12.7 years for Caucasian girls and 12.1 for Black girls. Menarche also occurs earlier for heavier girls and later for thinner, physically active girls.

A 32-year-old G0 woman comes to the office due to the inability to conceive for the last two years. She reports having been on oral contraceptives for eight years prior. She had menarche at age 14 and had irregular cycles about every three months until she started birth control pills, which made her cycles regular. In the last year, she has had about five cycles in total; her last menstrual period was six weeks ago. She is otherwise in good health and has not had any surgeries. She has no history of abnormal Pap smears or sexually transmitted infections. Her husband of four years is 35-years-old, and has a 10-year-old son from a previous marriage. She is 5 feet 4 inches tall and weighs 165 pounds. On general appearance, she is hirsute on the face and the abdomen. The rest of her exam is otherwise normal. In addition to weight loss and starting metformin, what is the most appropriate treatment for this patient's infertility problem? A. Laparoscopy B. Ovulation induction agents C. Intrauterine insemination D. In vitro fertilization E. Restart oral contraceptives

B. This patient has PCOS (polycystic ovarian syndrome) based on her history, signs and symptoms. It is most important for her to try to lose weight. Metformin and ovulation induction agents are the first-line of treatment for ovulatory dysfunction in PCOS patients. Since there is known ovulatory dysfunction and there is no reason to believe there are problems with semen analysis, IVF is not justified. There is no role for the laparoscopy in this patient. Although oral contraceptives would regulate her cycles, it is not indicated because she is trying to get pregnant.

A 27-year-old G0 woman presents to the clinic because of concerns that she has not been able to get pregnant for the last year. She has been married for two years and was using oral contraceptives, which she stopped a year ago to start a family. She is in good health and her only medication is a prenatal vitamin. She was hospitalized at age 19 for pelvic inflammatory disease. Her periods are regular, every 28 days with normal flow; her last period was two weeks ago. She has no history of abnormal Pap smears. Her husband is also healthy with no medical problems. She is 5 feet 4 inches tall and weighs 130 pounds. Her examination, including a pelvic exam, is completely normal. Which of the following is the most likely diagnostic test to find out the cause of her infertility? A. Hysteroscopy B. Hysterosalpingogram C. Progesterone level mid-cycle D. Clomiphene citrate challenge test E. Cervical mucous monitoring

B. This patient is having difficulty conceiving after trying for one year. Based on her history, the most likely underlying factor is tubal disease, as she has a history of being hospitalized for a pelvic infection, most likely pelvic inflammatory disease. This can cause adhesions and blockage of the tubes, which is best assessed with a hysterosalpingogram to evaluate the uterine cavity and tubes. After a single episode of salpingitis, 15% of patients experience infertility. Hysteroscopy will assess the uterine cavity and while sometimes used during a work up for infertility, it does not provide sufficient information about tubal patency. Progesterone levels, a Clomiphene challenge test or cervical mucous monitoring are used at times with infertility workups, but, in a young patient of normal BMI and with normal cycles, it is unlikely to find major ovulatory dysfunction.

A 36-year-old G0 woman presents due to increasing facial hair growth and irregular menstrual cycles. She has gained 40 pounds over the last three years. Her symptoms began three years ago and have gradually worsened. She has never been pregnant and is not currently on any medications. On physical exam, she is overweight with dark hair growth at the sideburns and upper lip. The pelvic exam is normal. Which of the following would you expect to find in this patient? A. Decreased luteinizing hormone levels B. Elevated free testosterone C. Decreased prolactin level D. Increased ovarian estrogen production E. Elevated 17-hydroxyprogesterone

B. This patient likely has polycystic ovarian syndrome (PCOS). PCOS patients have testosterone levels at the upper limits of normal or slightly increased. Free testosterone (biologically active) is elevated often because sex hormone binding globulin is decreased by elevated androgens. LH is increased in response to increased circulating estrogens fed by an elevation of ovarian androgen production. Insulin resistance and chronic anovulation are hallmarks of PCOS. Prolactin levels may be elevated in amenorrhea but are not elevated in patients with PCOS.

A 32-year-old G0 woman comes to the office due to the inability to conceive for last two years. She reports having been on oral contraceptives for eight years prior. She had menarche at age 14 and has had irregular cycles about every three months until she started oral contraceptives, which regulated her cycles. In the last year, she has had about five cycles in total; her last menstrual period was six weeks ago. She is otherwise in good health and has not had any surgeries. She has no history of abnormal Pap smears or sexually transmitted infections. She is 5 feet 4 inches tall and weighs 165 pounds. On general appearance, she is hirsute on the face and the abdomen. The rest of her exam is otherwise normal. Which of the following is most likely to help identify the underlying cause of this woman's infertility? A. Luteinizing hormone levels B. Testosterone levels C. Follicle stimulating hormone levels D. Thyroid function tests E. Progesterone levels

B. This patient most likely has PCOS (polycystic ovarian syndrome) based on her history of irregular cycles, her body habitus and hirsutism. Having normal cycles on the birth control pills (OCPs) supports the diagnosis as other causes, such as hypothyroidism, will not normalize the cycles on OCPs. Testosterone levels will be helpful to confirm the diagnosis, especially in the presence of hirsutism. Once a diagnosis is established, progesterone levels are helpful during medical treatment to check if the woman is ovulating. An increased LH/FSH ratio is observed to be elevated in PCOS patients but each test separately will not aid in the diagnosis.

A 42-year-old G0 woman presents to the office for a health maintenance examination. She reports that her menses have been irregular her entire life ever since menarche at age 15, occurring every 20-45 days. She is not sexually active and reports no other medical problems. She smoked for two years during her adolescence. She has a family history of cervical cancer affecting her mother at age 42. On examination, she is 5 feet 4 inches tall and weighs 180 pounds (BMI 31). She has noticeable hair growth on her upper lip and chin. The rest of her examination including a pelvic examination is normal. Compared to the general population, which of the following malignancies is she at increased risk for developing? A. Breast B. Endometrial C. Cervical D. Lung E. Colorectal

B. This patient most likely has polycystic ovarian syndrome (PCOS), with her clinical manifestation of oligo-menorrhea, obesity, and hirsutism. Because of the chronic unopposed estrogen exposure that accompanies women with PCOS, these individuals carry a higher risk of developing endometrial hyperplasia and cancer. Although obesity in postmenopausal women is associated with a higher risk of breast cancer, it does not increase the risk in premenopausal women. PCOS is considered to increase the risk of ovarian cancer. She does not have obvious risk factors for cervical cancer, lung, or colon cancer, but should be screened and counseled accordingly based on usual guidelines.

An 8-year-old female has been diagnosed with precocious puberty due to presence of menarche, Tanner stage III breasts and otherwise normal work-up for brain, adrenal and ovarian abnormalities. What is the most appropriate next step in the management of this patient? A. Depo-Provera B. GnRH agonist C. Danazol D. Estradiol E. Observation

B. True precocious puberty is manifested by premature secretion of GnRH hormone in a pulsatile manner. Once other causes of hormone production are ruled out, treatment would include GnRH agonist to suppress pituitary production of follicular-stimulating hormone and luteinizing hormone. Observation is acceptable if the precocious puberty is within a few months of the routinely expected puberty. The process should be treated if the bone age or puberty is advanced by several years.

A 45-year-old G2P2 woman comes to the office because of heavy and irregular menstrual periods. The heavy periods started three years ago and have gradually worsened in amount of flow over time. The periods are interfering with her daily activities. The patient has had two spontaneous vaginal deliveries, followed by a tubal ligation three years ago. On pelvic examination, the cervix appears normal and the uterus is normal in size without adnexal masses or tenderness. A urine pregnancy test is negative. TSH and prolactin levels are normal. Hemoglobin is 12.5 mg/dl. On pelvic ultrasound, she has a normal size uterus and a 2 cm simple cyst on the right ovary. Endometrial biopsy is consistent with a secretory endometrium; no neoplasia is found. What is the most likely diagnosis in this patient? A. Polycystic ovarian syndrome B. Mid-cycle bleeding C. Abnormal uterine bleeding D. Benign cystic teratoma E. Ovarian cancer

C. Abnormal uterine bleeding is a term used to describe uterine bleeding abnormalities. This term can encompass both structural causes (polyp, adenomyosis. Leiomyoma, or malignancy [or hyperplasia]) as well as non-structural causes (coagulopathies, ovulatory dysfunction, endometrial, iatrogenic or not classified). The acronym PALM-COEIN is a means for this classification. This patient had a complete workup, including TSH, Prolactin, pelvic ultrasound and endometrial biopsy, which were all normal. Mid-cycle bleeding at the time of ovulation is due to the drop in estrogen. Ovarian teratomas are not associated with abnormal menses. They typically present with abdominal or pelvic pain which may be associated with torsion. The 2 cm cyst is a functional cyst and is a common finding in ovulatory patients.

A 48-year-old G2P2 woman comes to your office because she has skipped her menstrual period for the past three months. She denies any menopausal symptoms. Review of symptoms and physical exam are unremarkable. What is the most likely diagnosis in this patient? A. Hypothyroidism B. Early pregnancy C. Perimenopause D. Premature ovarian failure E. Autoimmune disorder

C. Although there has been a decline in the average age of menarche with the improvement in health and living conditions, the average age of menopause has remained stable. The Massachusetts Women's Health Study reports that the average age of menopause is 51.3. This patient is most likely perimenopausal and will probably have more menstrual periods in the future. Although it is important to consider pregnancy and hypothyroidism, this patient's presentation is most consistent with perimenopause. Premature ovarian failure occurs before age 35.

A 23-year-old G0 woman presents to the office because she has not had any menses for four months. She has a long history of irregular menstrual cycles since menarche at age 14. She is in good health and is not taking any medications. She is sexually active with her partner of six months, and uses condoms for contraception. She is 5 feet 4 inches tall and weighs 170 pounds. On exam, she has noticeable hair growth on her upper lip and chin. The rest of her examination including a pelvic examination is normal. Her Beta-hCG is < 5 mIU/mL, and her prolactin and TSH levels are normal. In addition to recommending weight loss, what is the most appropriate next step in the management of this patient? A. Treatment with gonadotropin releasing hormone level (GnRH) agonist B. Treatment with clomiphene citrate C. Treatment with oral contraceptives D. Check progesterone levels E. Check cortisol levels

C. Oral contraceptives (OCPs) are the most appropriate treatment for this patient who most likely has the diagnosis of polycystic ovarian syndrome (PCOS). The constellation of findings support this clinical diagnosis (irregular cycles, obesity, and hirsutism). Because she is using condoms for contraception and is sexually active, OCPs would help regulate her cycles and further provide effective contraception. When she desires pregnancy, however, she will most likely need treatment for ovulation induction due to the anovulatory cycles as the leading cause of her oligomenorrhea. Clomiphene citrate is not indicated at this time.

A 36-year-old G1P1 woman comes to the office due to hair loss. She delivered a healthy infant girl three months ago. She is currently on a progestin-only oral contraceptive pill since she is breastfeeding. In the last month, she has noticed a large amount of hair on her brush each morning. Her father has male pattern baldness and her mother, who is postmenopausal, has had some thinning of her hair, as well. Testosterone and TSH levels are within the normal range. Which of the following is the most likely underlying cause for alopecia in this patient? A. Genetic predisposition B. Progesterone only pills C. High estrogen levels during pregnancy D. Stress during pregnancy and delivery E. Breastfeeding

C. Postpartum telogen effluvium (hair loss) affects 40-50% of women postpartum. High estrogen levels in pregnancy increase the synchrony of hair growth. Therefore, hair grows in the same phase and is shed at the same time. Occasionally, this can result in significant postpartum hair loss at 1 to 5 months postpartum with 3 months after delivery being most common time. In the non-pregnant state, asynchronous hair growth occurs such that a portion of hair is in one of the three hair growth cycles at all times. While genetic predisposition and stress can be causes of hair loss, they are unlikely explanation in this post partum woman.

A 23-year-old G0 woman with severe dysmenorrhea that is unresponsive to non-steroidal anti-inflammatory agents and oral contraceptives is taken to the operating room for a laparoscopy. Blue-black powder burn lesions are seen in the pelvis. A biopsy is performed and sent to pathology. Which of the following pathologic findings would you expect to see in this patient? A. Hyperplastic overgrowth of endometrial glands/stroma B. Decidual effect in the endometrium C. Endometrial glands/stroma and hemosiderin-laden macrophages D. Invasion of endometrial glands into the myometrium E. Well-circumscribed, non-encapsulated myometrium

C. The lesions described are classic for endometriosis. One would therefore expect to see endometrial glands/stroma with hemosiderin-laden macrophages. Hyperplastic overgrowth of endometrial glands/stroma is consistent with endometrial polyps. Decidual effect on the endometrium are seen during pregnancy. Invasion of endometrial glands into the myometrium is seen with adenomyosis. Well-circumscribed, non-encapsulated myometrium is consistent with myomas.

A 41-year-old G2P2 woman presents with menstrual pain, menorrhagia, irregular periods and intermenstrual bleeding. She describes the pain as pressure and cramps. Ibuprofen improves the pain, but does not entirely eliminate the discomfort. Pelvic examination reveals a 14-week size uterus with irregular masses within the uterus. Pelvic ultrasound confirms the diagnosis of fibroids. What is the most appropriate next step in the management of this patient? A. CA125 assay B. CT scan of the pelvis C. Endometrial biopsy D. GnRH agonist E. Hysterectomy

C. This patient has classic symptoms of leiomyomata, including menorrhagia. An endometrial biopsy should be performed on all women over age 40 with irregular bleeding to rule out endometrial carcinoma. The CA125 assay measures the level of CA125 in the blood and is increased in some types of cancer, including ovarian cancer or other conditions. This non-specific marker is not indicated in this patient. A CT scan of the pelvis is also not indicated. A simple pelvic ultrasound could be used to help confirm the clinical diagnosis. GnRH agonist and hysterectomy are not used until the diagnosis of leiomyomata uteri is confirmed. Her desire for future fertility should be discussed.

A 37-year-old G2P1011 woman comes to the clinic with her husband due to the inability to conceive for the last year. She reports being in good health and not having problems with her prior pregnancy two years ago, except for some postpartum depression for which she was placed on imipramine and which she continues to take. She took birth control pills after her pregnancy and stopped one year ago, when she began trying to conceive. Her periods were regular on the pills, but have been irregular since she stopped taking them. She has no history of sexually transmitted infections or abnormal Pap smears. Her husband is also healthy and he fathered their first child. Her physical examination is completely normal. Laboratory tests show: Results Normal Values TSH 2.1 mIU/ml 0.5-4.0 mIU/ml Free T4 1.1 ng/dl 0.8-1.8 ng/dl Prolactin 60 ng/ml <20 ng/ml FSH 6 mIU/ml 5-25 mIU/ml LH 4 mIU/ml 5-25 mIU/ml What is the most appropriate next step in the management of this patient's subfertility? A. Begin bromocriptine B. Ovulation induction with clomiphene citrate (Clomid) C. Wean off imipramine D. Perform a visual field examination E. Obtain a brain MRI

C. This patient has hyperprolactinemia due to imipramine. The patient has to be weaned off imipramine (instead of abrupt discontinuation to minimize withdrawal symptoms) and placed on a more appropriate medication. Once she is off imipramine and the cause of her elevated prolactin levels is confirmed, her normal menses should resume. Although MRI of the brain would be a reasonable step, it would be premature, and visual field examination does not aid in the diagnostic work-up. It would be premature to obtain an MRI or begin bromocriptine without this intermediate step. An endometrial biopsy is not indicated at this point, especially since the patient had normal cycles on OCPs. Although Clomid is used to help with ovulatory dysfunction, the hyperprolactinemia must be addressed first.

A 45-year-old G3P3 woman comes to the office because she has been unable to conceive for the last two years. She is healthy and has three children, ages 10, 12 and 14, whom she conceived with her husband. She used a copper IUD after the birth of her last child and had it removed two years ago, hoping to have another child. She has no history of sexually transmitted infections or abnormal Pap smears. Her cycles are regular every 28 to 32 days. She is not taking any medications. She has been married for the last 16 years, and her husband is 52-years-old and in good health. Her physical examination, including a pelvic exam, is completely normal. Which of the following is the most appropriate next step in the management of this patient? A. Hysteroscopy B. Hysterosalpingogram C. Clomiphene challenge test D. Sperm penetration assay E. Basal body temperatures for six months

C. This patient, most likely, has decreased ovarian reserve due to her age. A clomiphene challenge test, which consists of giving clomiphene citrate days five to nine of the menstrual cycle and checking FSH levels on day three and day 10, will help determine ovarian reserve. This will help counsel the patient on appropriate options to have a child, as most women will not be able to conceive at this age and would not be good candidates for ovarian stimulation or IVF. Even though this patient had an IUD in the past, there is no reason to believe that this contributed to her inability to conceive, as IUDs do not cause infertility and she has no risk factors for tubal disease. Having her keep basal body temperatures for six months would be a waste of time for this patient, who is already 45. A semen analysis or sperm penetration assay is not necessary as a first step as she was able to conceive from her husband previously without problems. Most likely this patient will have to use a donor egg if she wants to carry the pregnancy herself.

A 7-year-old female is undergoing evaluation for vaginal bleeding. On physical examination, she has Tanner stage III breasts, tall stature and an otherwise normal examination. An MRI of the brain and a pelvic ultrasound are normal. LH and FSH levels are in the pubertal levels and she has normal DHEAS and androgen levels. What is the most likely diagnosis in this patient? A. Pituitary adenoma B. Congenital adrenal hyperplasia C. True precocious puberty D. McCune Albright Syndrome E. Ovarian neoplasm

C. True precocious puberty is a diagnosis of exclusion where the sex steroids are increased by the hypothalamic-pituitary-gonadal axis, with increased pulsatile GnRH secretion. CNS abnormalities associated with precocious puberty include the following: tumors (e.g., astrocytomas, gliomas, germ cell tumors secreting human chorionic gonadotropin [hCG]); hypothalamic hamartomas; acquired CNS injury caused by inflammation, surgery, trauma, radiation therapy, or abscess; or congenital anomalies (e.g. hydrocephalus, arachnoid cysts, suprasellar cysts). These conditions are not likely in the presence of a normal work-up in this patient. Congenital adrenal hyperplasia usually presents in the neonatal period and is associated with ambiguous genitalia. McCune Albright Syndrome is characterized by premature menses before breast and pubic hair development. An ovarian neoplasm is unlikely with a normal pelvic ultrasound.

A 28-year-old G0 woman comes to the office for preconception counseling and the inability to conceive for one year. She and her husband of three years are both in good health. She has normal cycles every 28-33 days. She has intercourse about once a month, depending on her schedule. She is an airline pilot and travels a lot. Her examination is normal. She asks about when to best have intercourse during her cycle to maximize her chances of pregnancy. What is the most appropriate advice to give her? A. Keep basal body temperatures and try to attempt intercourse immediately after the rise in body temperature B. Best to attempt intercourse after she is done with her menses C. Use ovulation predictor kits and attempt intercourse after it turns positive D. Take a leave from her work so she can have intercourse three times a week until she gets pregnant E. Attempt intercourse on day 18 of her cycle

C. Women are most fertile during the middle of their cycle when they are ovulating. Assuming normal cycles every 28 days, a woman is most likely to ovulate on day 14. Since sperm can live for up to three days, intercourse up to three days before ovulation can still result in pregnancy. Since this patient has cycles that vary in length, she can best tell when she is ovulating by using an ovulation predictor kit. The basal body temperature charts tell when a patient ovulated retrospectively, so it cannot be used to time intercourse to conceive, as the egg is only viable for about 24 hours. Although having intercourse more frequently will increase her likelihood of conceiving, it is not a practical solution for a working person to stop their work in order to conceive.

A 42-year-old G2P2 woman complains of bloating, mood swings and irritability the week prior to her menses. She is convinced that something is wrong with her hormone levels. In addition to a complete physical examination, which of the following diagnostic tools would provide information to accurately determine the diagnosis? A. Pelvic ultrasound B. Estradiol level C. CAGE questionnaire D. Prospective symptom calendar E. Mini mental status examination

D. A calendar of symptoms can clarify if there is a cyclic or constant nature of the symptoms. Often women will mistakenly attribute their symptoms to their menstrual cycle. Different self-reporting scales have been written to assist patients track their symptoms. Because she is menstruating regularly, there is no role for obtaining serum hormone levels.

A 41-year-old G3P3 woman reports heavy menstrual periods occurring every 26 days lasting eight days. The periods have been increasingly heavy over the last three months. She reports soaking through pads and tampons every two hours. She has a history of three uncomplicated spontaneous vaginal deliveries and a tubal ligation following the birth of her last child. On pelvic examination, the cervix appears normal and the uterus is normal in size. Which of the following tests or procedures would be most useful in further evaluation of this patient's complaint? A. Follicle stimulating hormone level B. Prolactin level C. Coagulation studies D. Pelvic ultrasound E. Hysteroscopy

D. A pelvic ultrasound would image the endometrium and assess for endometrial pathology such as polyps or submucosal fibroids. In the absence of menopausal symptoms, FSH is unlikely to be helpful. The patient is unlikely to have a coagulation disorder, as she has had three spontaneous vaginal deliveries without postpartum hemorrhage. Hysteroscopy is more invasive than an ultrasound as a first step and would not be helpful if the cause of abnormal bleeding is myometrial pathology such as intramural and subserosal fibroids or adenomyosis. Hyperprolactinemia is found with prolactin-secreting adenomas associated with amenorrhea.

A 24-year-old G0 woman comes into the office because she has not had her menses for six months. She is in good health and not taking any medications. She is not sexually active. She does well in graduate school, despite her demanding new program. Her height is 5 feet 6 inches and her weight is 104 pounds. Her vital signs are normal. Her physical examination, including a pelvic examination, is completely normal. What is the most likely reason for her amenorrhea? A. Ovarian dysfunction B. Thyroid disease C. Premature ovarian failure D. Hypothalamic-pituitary dysfunction E. Pregnancy

D. Anorexia nervosa or significant weight loss may cause hypothalamic-pituitary dysfunction that can result in amenorrhea. A lack of the normal pulsatile secretion of gonadotropin releasing hormone (GnRH) leads to a decreased stimulation of the pituitary gland to produce follicle stimulating hormone (FSH) and luteinizing hormone (LH). This leads to anovulation and amenorrhea. Although testing for thyroid dysfunction may be indicated, she has no other symptoms to suggest thyroid disease. While ovarian dysfunction/failure, premature ovarian failure and pregnancy cause amenorrhea, they are unlikely in this case.

A 54-year-old G4P4 woman who has been menopausal for four years recently underwent a total vaginal hysterectomy and bilateral salpingo-oophorectomy for vaginal prolapse. She comes in for a postoperative check up and complains of hot flashes and wonders why she is experiencing menopause again. Which of the following most likely explains why she is experiencing these symptoms? A. Increased postoperative liver metabolism B. Decreased adrenal estrogen production C. Removal of an occult estrogen-producing tumor D. Decreased circulating androgens E. Cessation of ovarian estrogen production

D. Estrogen production by the ovaries does not continue beyond menopause. However, estrogen levels in postmenopausal women can be significant due to the extraglandular conversion of androstenedione and testosterone to estrogen. This conversion occurs in peripheral fat cells and, thus, body weight has been directly correlated with circulating levels of estrone and estradiol. Since menopausal ovaries are known to continue production of androgens, surgical removal of postmenopausal ovaries may result in the resurgence of menopausal symptoms from the abrupt drop in circulating androgens.

A 26-year-old G0 woman comes to the office due to irregular menses since menarche, worsening for the last six months. The patient has noted increasing hair growth on her chin and most recently hair growth on her chest, requiring that she shave periodically. No one in her family has hirsutism. On exam, you also notice acne on her chin, acanthosis nigricans and temporal balding. Her serum testosterone is elevated. You suspect hyperthecosis. Which of the following might also be associated with this condition? A. Hyperthyroidism B. Hyperprolactinemia C. Atrophic changes of external genitalia D. Deepening of the voice E. Hyperparathyroidism

D. Hyperthecosis is a more severe form of polycystic ovarian syndrome (PCOS). It is associated with virilization due to the high androstenedione production and testosterone levels. In addition to temporal balding, other signs of virilization include clitoral enlargement and deepening of the voice. Hyperthecosis is more difficult to treat with oral contraceptive therapy. It is also more challenging to achieve successful ovulation induction. Hyperthyroidism and hyperparathyroidism are not typically associated with hyperthecosis. Hyperprolactinemia is typically associated with amenorrhea and does not cause hirsutism.

A 42-year-old G4P4 woman presents for management of suspected adenomyosis. She had a tubal ligation four years ago. A pelvic examination shows an enlarged, soft, boggy uterus. A pregnancy test is negative and she is mildly anemic. An ultrasound shows an enlarged uterus with no fibroids. The patient desires definitive treatment for this condition. What is the most appropriate next step in her management? A. Continuous estrogen/progestin therapy B. Endometrial ablation C. GnRH agonist D. Hysterectomy E. Insertion of a levonorgestrel containing intrauterine system

D. Hysterectomy is nearly 80% effective in eliminating pain and abnormal bleeding, if she is willing to undergo surgery. Gonadotropin releasing agents are the first choice for medical therapy for the pain, but the problem is that the adenomyosis seems to recur after discontinuing the therapy. Endometrial ablation and insertion of a levonorgestrel-containing intrauterine system are options in women who decline hysterectomy or desire to maintain fertility. For abnormal bleeding problems and desire for uterine conservation, a progesterone intrauterine contraceptive device can also be used to improve irregular bleeding. Hysteroscopic endometrial ablation can be a treatment for adenomyosis.

A 34-year-old G2P2 woman presents with inter-menstrual bleeding for one year. The bleeding typically occurs two weeks after her menses and last two to three days. The symptoms began one year ago and the bleeding has not changed recently. She is currently taking oral contraceptives. On pelvic examination, the cervix appears normal and the uterus is normal in size and shape. Her urine pregnancy test is negative; an endometrial biopsy is negative for hyperplasia. Which of the following tests or procedures would be indicated for further work-up? A. Prolactin level B. Progesterone level C. Hysterosalpingogram (HSG) D. Pelvic ultrasound E. Colposcopy

D. Intermenstrual bleeding is frequently caused by structural abnormalities of the endometrial cavity, such as myomas, polyps or malignancy. An ultrasound would be helpful as the next step in diagnosis. Although an HSG might reveal structural abnormalities, it is too invasive as the next step. A colposcopy would not be helpful in the diagnosis, nor would obtaining a Prolactin level, as it would be indicated for the evaluation of anovulatory bleeding. Progesterone levels are not helpful in a patient on oral contraceptives.

A 13-year-old female is brought to the physician for increasingly severe abdominal pain. The pain is now constant and mildly uncomfortable, but every month she has a week when it is more severe. She has Tanner stage II breasts and pubic hair development. On genital examination, there is a bluish mass pushing the labia open. What is the most likely cause of this patient's abdominal pain? A. Turner's syndrome B. Transverse vaginal septum C. Isolated atresia of the cervix D. Imperforate hymen E. Synechiae of the uterine cavity

D. Lower genital tract malformations occur in 1 in 10,000 females and are most commonly an imperforate hymen where the genital plate canalization is incomplete. Amenorrhea and abdominal pain are also associated with isolated atresia of the vagina or cervix. The menstrual blood will collect in the vagina and uterus causing pain. Treatment involves surgical correction. When a transverse vaginal septum is present, a normal vaginal opening with a short blind vagina and pelvic mass may be located above the level of the obstruction found on exam. Asherman's syndrome is associated with secondary amenorrhea resulting from intrauterine scarring/synechiae.

A 32-year-old G0 woman presents with amenorrhea for the last three months. She has a long history of irregular cycles, 26 to 45 days apart, for the last two years. She is otherwise in good health and is not taking any medications. She is sexually active with her husband and uses condoms for contraception. She is 5 feet 4 inches tall and weighs 140 pounds. On exam, she has a slightly enlarged, non-tender uterus. There are no adnexal masses. Which of the following is the most appropriate test to obtain in this patient? A. Thyroid stimulating hormone (TSH) B. Progesterone and estrogen C. Follicle stimulating hormone (FSH) D. Urine pregnancy test E. Pelvic ultrasound

D. Pregnancy is the most common cause of amenorrhea. It is important to consider it early in the workup to avoid unnecessary tests, procedures and treatments that may be contraindicated during pregnancy. Although the patient has a history of irregular cycles and is using condoms for contraception, it is important to first rule out pregnancy before initiating further work-up.

A 37-year-old G1P1 woman has experienced symptoms of depression and difficulty concentrating the week prior to her menstrual period for the last three years, since her tubal ligation. She kept a symptom diary for three months revealing symptoms clustered around her menstrual cycle. She was diagnosed with premenstrual syndrome and began a regular exercise routine with dietary modifications, but only noticed mild relief in her symptoms. Work-up is otherwise unremarkable. Which of the following will most likely alleviate her symptoms? A. Evening primrose oil B. Ginkgo C. Progesterone cream D. Fluoxetine hydrochloride E. Levothyroxine sodium

D. Selective serotonin reuptake inhibitors increase the amount of active serotonin in the brain and have been found to be effective in alleviating PMS and PMDD symptoms. Patients can take the medication either every day or for 10 days during the luteal phase. Progesterone cream will not help her symptoms. Herbal therapies such as evening primrose oil and ginkgo have not been shown to improve PMDD symptoms. Symptoms due to hypothyroidism would be present throughout her cycle.

A 32-year-old G2P2 woman complains of depression, weight gain and premenstrual bloating. She has suffered from these symptoms for 18 months and they have not responded to dietary changes and avoidance of alcohol and caffeine. Her only medications are multivitamins and herbs to increase her energy. She is very concerned about fatigue that often interferes with caring for her two children. A prospective symptom diary completed by the patient indicates mood symptoms, fatigue and bloating almost every day of the past two months, and regular menstrual cycles accompanied by breast tenderness. She denies feelings of wanting to hurt herself or others. Physical examination is unremarkable. Which of the following conditions is the most likely explanation for this patient's symptoms? A. Panic disorder B. Anxiety disorder C. Anemia D. Hypothyroidism E. Premenstrual dysphoric disorder

D. Symptoms of hypothyroidism can mimic typical symptoms of PMS, but symptoms occur more constantly throughout the cycle. Diagnosis involves complete work-up to rule out medical illnesses, including hypothyroidism. Although fatigue can be associated with anemia, her presentation is not consistent with this diagnosis.

A mother brings her 16-year-old daughter to the doctor because she has not begun menses. She performs poorly in school because of dyslexia. On physical examination, she is 4 feet 11 inches tall, 100 pounds and has Tanner stage I breast and pubic hair growth. Her chest is broad, breast nipples are widely spaced and her neck is thickened. No genital tract abnormalities are noted on exam. Which of the following is the most likely cause of her delayed sexual maturation? A. Partial deletion of the long arm of the X chromosome B. Down Syndrome C. Noonan Syndrome D. Turner Syndrome E. Rokitansky-Kuster-Hauser Syndrome

D. The genetic defect of Turner syndrome is the absence of one of the X chromosomes. These females have failure to establish secondary sexual characteristics, short stature and characteristic physical features: pterygium colli, shield chest and cubitus valgus. Partial deletions of the long arm of the X chromosome also cause premature ovarian failure. The average age of puberty in females with Down syndrome is not significantly different than normal females. Clinically, patients with Noonan syndrome typically have normal puberty and fertility. They may have short stature, webbed neck, heart defects, and abnormal faces. Individuals with Noonan syndrome have a normal karyotype. Rokitansky-Kuster-Hauser Syndrome causes vaginal and uterine agenesis and is not suspected in this case due to the normal pelvic exam findings.

A 33-year-old G0 woman presents with amenorrhea for the past 12 months. She also reports a recent onset of dyspareunia, causing her to feel anxious about having intercourse. She had menarche at age 15. Her cycles were normal until two years ago when she began skipping menses. She is otherwise in good health. She is 5 feet 4 inches tall and weighs 130 pounds. Her physical examination is completely normal. TSH and prolactin levels are normal. Urine pregnancy test is negative. What is the most likely cause of this patient's amenorrhea? A. Psychogenic B. Genital tract outflow obstruction C. Asherman's syndrome D. Premature ovarian failure E. Pituitary adenoma

D. The patient's symptom of dyspareunia is likely caused by vaginal dryness, which is associated with estrogen deficiency. Hypergonadotropic amenorrhea is the result of ovarian failure or follicular resistance to gonadotropin stimulation. The history, physical exam and labs make the other possibilities less likely: psychogenic disorder (no chronic anxiety or anorexia nervosa), outflow obstruction (previously had periods), Asherman's syndrome (no history of pregnancy or intrauterine procedures), or a pituitary tumor (normal labs).

A 22-year-old G0 woman presents with painful menstruation that limits her activities each month. She describes the pain as spasmodic occurring on days one to three of bleeding since her cycles began. Other symptoms include nausea, nervousness, diarrhea, and headache. Her physical exam is normal with a soft, non-tender abdomen. Bimanual exam reveals a fixed uterus with uterosacral ligament nodularity. There are no adnexal masses noted. Which of the following is the most likely diagnosis in this patient? A. Premenstrual syndrome B. Premenstrual dysphoric disorder C. Primary dysmenorrhea D. Secondary dysmenorrhea E. Adenomyosis

D. The physical examination in patients with primary dysmenorrhea is normal. There should not be any palpable abnormalities on abdominal, speculum, pelvic, bimanual, and rectal examinations. The restricted uterine motion found on exam suggests the possibility of endometriosis or pelvic scarring from inflammation or adhesions. These conditions must be considered in establishing the etiology of her diagnosis. Childbearing does not affect the occurrence of either diagnosis. Although the patient's symptoms, including the associated symptoms, timing of initial onset, and cyclic nature of her pain are consistent with primary dysmenorrhea, the finding on physical examination makes secondary dysmenorrhea the likely diagnosis.

A 35-year-old G0 Asian woman presents with irregular menses and hirsutism of three months duration. The patient has no family history of hirsutism. On exam, the patient was noted to have terminal hair growth on her chest and recently had laser treatment to remove similar hair on her chin. Her total testosterone is 76 ng/dl (normal) and her DHEAS is 1500µg/dl (elevated). Which of the following is the most likely diagnosis in this patient? A. Pituitary adenoma B. Ovarian tumor C. Cushing's syndrome D. Adrenal tumor E. Idiopathic

D. The short duration of symptoms and the significantly elevated DHEAS support the diagnosis of an adrenal tumor as the etiology of the patient's symptoms. In addition, the patient is Asian and is less likely to have a predisposition to idiopathic hirsutism. Asians with polycystic ovarian syndrome are less likely to present with overt hirsutism than other ethnic groups.

A 58-year-old G3P1 woman presents to your office for her a health maintenance examination. She became menopausal at age 54. Her past medical history is significant for angina. She experienced a hip fracture 14 months ago when she tripped and fell while running after her grandson. She has not had any surgeries. She takes no medications and has no known drug allergies. She smokes 10 cigarettes a day and drinks a glass of red wine at dinner. Her father was diagnosed with colon cancer at the age of 72. Physical exam revealed a blood pressure of 120/68, pulse of 64, and BMI of 22. Her heart, lung, breast and abdominal exams were normal. Pelvic exam was consistent with vaginal atrophy and a small uterus. There was no adnexal tenderness and no masses were palpated. In addition to obtaining a bone mineral density scan, what is the next step in the management plan for this patient? A. Repeat bone mineral density in one year B. Repeat bone mineral density at age 65 C. Begin hormone replacement therapy D. Begin treatment with bisphosphonates E. Test for the presence of biochemical bone markers in the blood

D. This patient has many of the major risk factors for osteoporosis including history of fracture as an adult, low body weight and being a current smoker. Patients who already have had an osteoporotic fracture may be treated on this basis alone. Prior to beginning treatment with bisphosphonates, a bone mineral density (BMD) should be documented and repeated at two-year intervals to monitor treatment. DEXA is the test of choice for measuring BMD. A nuclear medicine bone scan may be useful to rule out a pathologic fracture from metastatic disease. General recommendations for the prevention of osteoporosis include eating a balanced diet that includes adequate intake of calcium and vitamin D, regular physical activity, avoidance of heavy alcohol consumption, and smoking cessation. Bone markers are used in research but are not yet a reliable predictor of BMD. Hormone replacement therapy is not recommended long term for disease prevention especially in patients with cardiovascular disease.

A 27-year-old G0 woman presents to the clinic because of concerns that she has not been able to get pregnant for the last year. She has been married for two years ago and was using birth control pills for contraception. She stopped using birth control pills when she decided to start a family one year ago. She is in good health and her only medication is a prenatal vitamin. Her periods are regular, every 28 days, with normal flow; her last period was two weeks ago. She has no history of sexually transmitted infections and no abnormal Pap smears. Her husband is also healthy with no medical problems. She is 5 feet 4 inches tall and weighs 130 pounds. Her examination, including a pelvic exam, is completely normal. Laboratory results show normal thyroid function tests and normal prolactin level. What is the most appropriate next step in the management of this patient? A. Reassurance and observation B. Perform a pelvic ultrasound C. Order a hysterosalpingogram D. Order a semen analysis E. Recommend a diagnostic laparoscopy

D. This patient has primary infertility, since she has not been able to conceive for one year. She does not appear to have underlying pathology to explain why she has not conceived, and her husband's semen has not yet been examined. The male factor plays a role in about 35% of infertility cases. A pelvic ultrasound is unlikely to add any information, as the patient has normal cycles and normal exam. Although a hysterosalpingogram might be ordered in the future, the male factor needs to be ruled out first, as it is less invasive to perform. Even though this patient had been on birth control pills previously, this should not be affecting her fertility a year later. In patients who use OCPs for prolonged periods, there might be a few months delay in returning to normal fertility.

A 37-year-old G1P1 woman suffers from severe mood swings the week before her menstrual cycle. The mood swings resolve after she stops bleeding. You diagnose her with premenstrual syndrome (PMS) after obtaining further history and a normal examination. In addition to exercise, which of the following might be suggested to help decrease this patient's symptoms? A. Folic acid B. Ginkgo C. Fish oil D. Vitamin B6 E. Potassium

D. Vitamin deficiency of A, E and B6 have been associated with an increase in PMS. Replacement of these vitamins might improve PMS symptoms and avoid further medical therapy.

A 22-year-old G0 woman presents with five months of amenorrhea since discontinuing her oral contraceptive pills. She had been on the pill for the last six years and had normal menses every 28 days while taking them. She is in good health and not taking any medications. She is 5 feet 4 inches tall and weighs 140 pounds. Her examination, including a pelvic examination, is normal. Which of the following historical elements would be most useful in determining the cause of amenorrhea in this patient? A. Age at first intercourse B. History of sexually transmitted infections C. Parity D. History of oligo-ovulatory cycles E. Recent history of weight loss

D.Since most women resume normal menstrual cycles after discontinuing oral contraceptive pills (OCPs), they are not usually considered the cause of the amenorrhea. A history of irregular cycles prior to pill use may increase the risk of amenorrhea upon discontinuation. This is sometimes referred to as "post pill amenorrhea." A complete work-up should be performed to properly find the cause. Although the other historical elements are all important components of a complete gynecological history, they are not helpful to find the etiology of amenorrhea in this patient. Significant weight loss might cause amenorrhea; however, this patient still has normal body mass index, which makes it unlikely cause of amenorrhea.

A 35-year-old G0 woman presents with irregular menstrual periods occurring every six to twelve weeks with occasional inter-menstrual bleeding. Currently, she has been bleeding daily for the last four weeks. She reports that her periods have always been irregular, but have become more so with heavier flow and cramping in the last year. She is sexually active with one partner. On physical exam, she is morbidly obese with no abnormalities detected on pelvic exam. Which of the following is the most appropriate next step in the management of this patient? A. Luteinizing hormone level (LH) B. Follicle stimulating hormone level (FSH) C. Testosterone level D. Pelvic CT E. Endometrial biopsy

E. Endometrial biopsy should be performed to rule out endometrial hyperplasia or carcinoma given the history of irregular bleeding, coupled with the increased risk of these diagnoses in morbidly obese patients. While an ultrasound may be helpful, a pelvic CT is not useful in the workup for potential endometrial neoplasia. LH and FSH levels would not aid in the diagnostic workup and testosterone levels would not be useful, unless signs of hirsutism or virilization are present.

An 18-year-old G0 woman comes to the office due to vaginal spotting for the last two weeks. Her menstrual periods were regular until last month, occurring every 28-32 days. Menarche was at age 13. She started oral contraceptives three months ago. On pelvic examination, the uterus is normal in size, slightly tender with a mass palpable in the right adnexal region. No adnexal tenderness is noted. Which of the following tests is the most appropriate next step in the management of this patient? A. Endometrial biopsy B. Pelvic MRI C. Pelvic ultrasound D. Abdominal CT Scan E. Urine pregnancy test

E. It is vitally important to rule out pregnancy in the evaluation of abnormal uterine bleeding. Pelvic ultrasound could be considered as a next step if the pregnancy test is negative in order to evaluate the adnexal finding. Abdominal CT or MRI would not be performed in this patient unless advanced adnexal pathology was found on pelvic sonography. Endometrial biopsy would rarely be indicated in a teen with abnormal bleeding, unless morbidly obese and anovulatory.

A 49-year-old G1P1 woman comes to your office for menopause counseling. She has been experiencing severe sleep disturbances and night sweats for the past four months. She would like to begin hormone therapy, but is concerned because she has elevated cholesterol levels for which she takes medication. You explain to her that hormone therapy has the following effect on a lipid/cholesterol profile: A. Both HDL and LDL levels increase B. Both HDL and LDL levels decrease C. HDL and LDL levels are unaffected D. HDL levels increase and LDL levels are unaffected E. HDL levels increase and LDL levels decrease

E. Recent data have confirmed the overall positive effects of hormone therapy on serum lipid profiles. The most important lipid effects of postmenopausal hormone treatment are the reduction in LDL cholesterol and the increase in HDL cholesterol. Estrogen increases triglycerides and increases LDL catabolism, as well as lipoprotein receptor numbers and activity, therefore causing decreased LDL levels. Hormones inhibit hepatic lipase activity, which prevents conversion of HDL2 to HDL3, thus increasing HDL levels. Hormone therapy is not currently recommended for the primary prevention of heart disease.

A 37-year-old G3P3 woman complains of severe premenstrual symptoms for the past two years. She finds her mood swings and irritability troubling and requests a hysterectomy, as she thinks that this procedure will alleviate her symptoms. Past medical history is only remarkable for high cholesterol and her physical examination, including pelvic examination, is normal. The patient's physician does not recommend a hysterectomy. Which of the following is the most likely explanation for the physician's recommendation not to perform a hysterectomy in this patient? A. An endometrial ablation would be preferable B. Past medical history C. Influence of thyroid hormone on symptoms D. Influence of adrenal gland on symptoms E. Influence of ovaries on symptoms

E. The patient's mood swings are influenced by the hormonal shifts controlled by the hypothalamic-pituitary-ovarian axis. A hysterectomy or endometrial ablation would only resolve the menstrual bleeding component of this patient's symptoms, and have no effect on the hormonal production of the ovaries. Removal of her ovaries may improve her symptoms but would increase her risk for future problems, including osteoporosis and menopausal symptoms. A bilateral oophorectomy would be the last option for this patient as many medical options are available to treat her symptoms.

A 23-year-old G0 woman comes to the clinic because she is interested in becoming pregnant. She is in good health; however, she has not had any menses for the last two years. She had menarche at age 15, had normal periods until three years ago, when she started having periods irregularly every three months until it stopped two years ago. She has no history of pelvic infections or abnormal Pap smears. She exercises every day by running and has run four marathons in the last three years. She is 5 feet 10 inches tall and weighs 115 pounds. Her examination including a pelvic exam is normal. Laboratory results show: Results Normal Values TSH 3.5 mIU/ml 0.5-4.0 mIU/ml Free T4 0.9 ng/dl 0.8-1.8 ng/dl Prolactin 10 ng/ml <20 ng/ml FSH 6 mIU/ml 5-25 mIU/ml LH 4 mIU/ml 5-25 mIU/ml BHCG 2 mIU/ml <5 mIU/ml What is the most appropriate next step in the evaluation of this patient? A. Check cortisol levels B. Order a brain MRI C. Obtain a pelvic ultrasound D. Check testosterone levels E. Check estrogen levels

E. This patient most likely has exercise-induced hypothalamic amenorrhea, which is characterized by normal FSH and low estrogen levels. The other studies will not help determine the diagnosis. The best treatment is to encourage the patient to gain weight by decreasing exercise and increasing caloric intake. If her menses fail to resume, she may be treated with exogenous gonadotropins (LH and FSH) to help her conceive. Clomiphene citrate tends not to work as well, due to the baseline hypoestrogenic state.

A 22-year-old G0 woman presents with hirsutism which has been present since menarche. She states that she has laser treatments done to remove the hair on her chin every couple of months, and was wondering if there are additional treatments which might help her. She is otherwise in good health. She has normal menstrual cycles every 28 days. She is sexually active and uses birth control pills for contraception. The patient is adopted and has no information about family history. She is 5 feet 4 inches tall and weighs 125 pounds. On examination, the patient was noted to have terminal hair growth on her chest. Her TSH, Prolactin, total testosterone, DHEAS, 17-hydroxyprogesterone levels are normal. Which of the following is the most likely underlying etiology for the hirsutism in this patient? A. Polycystic ovarian syndrome B. Side effects of the oral contraceptives C. Cushing's syndrome D. Adrenal tumor E. Idiopathic hirsutism

E. This patient most likely has idiopathic hirsutism. She has no other clinical signs of polycystic ovaries, such as irregular cycles or obesity. Normal laboratory values rule out other pathogenic causes of hirsutism, such as Cushing's syndrome or adrenal tumor. Oral contraceptives are actually used for the treatment of hirsutism because they establish regular menses and lower ovarian androgen production. Additionally, they cause an increase in SHBG (sex hormone binding globulin) which allows more testosterone to be bound and unavailable at the hair follicle.

A 42-year-old G2P2 woman undergoes a hysterectomy for definitive treatment of her dysmenorrhea and large uterine fibroids. The uterus is sent to pathology. Which of the following would confirm the diagnosis of fibroids? A. Hyperplastic overgrowth of endometrial glands/stroma B. Decidual effect in the endometrium C. Endometrial glands/stroma and hemosiderin-laden macrophages D. Invasion of endometrial glands into the myometrium E. Well-circumscribed, non-encapsulated myometrium

E. Well-circumscribed, non-encapsulated myometrium confirms the diagnosis of fibroids. Hyperplastic overgrowth of endometrial glands/stroma is consistent with endometrial polyps. Decidual effect on the endometrium are seen during pregnancy. Invasion of endometrial glands into the myometrium is seen with adenomyosis.


Kaugnay na mga set ng pag-aaral

Ch. 6: Recognizing, Analyzing, and Constructing Arguments

View Set

Chapter 4: Managing Marketing Information to Gain Customer Insights

View Set

chapter 8 - articulations (espinoza)

View Set

Ch. 26 Deceptive Sales Practices

View Set

Performance Appraisal Rating Errors

View Set